wu :: forums (http://www.ocf.berkeley.edu/~wwu/cgi-bin/yabb/YaBB.cgi)
general >> wanted >> intersection of embedded closed sets
(Message started by: MonicaMath on Sep 17th, 2009, 11:55am)

Title: intersection of embedded closed sets
Post by MonicaMath on Sep 17th, 2009, 11:55am
Hi,

I need to prove that?

if {A_k}, k=1,..., infinity,  is a collection of nonempty embedded  closed sets of real numbers in decreasing order with A_j is bounded for one j, then :

the intersection is nonempty ??

Title: Re: intersection of embedded closed sets
Post by Eigenray on Sep 17th, 2009, 6:12pm
Are you familiar with the open cover definition of compactness?

If the intersection were empty, we would have

Aj = http://www.ocf.berkeley.edu/~wwu/YaBBImages/symbols/cup.gifk>j  Uk,

where Uk = Aj \ Ak is open in Aj.  Since Aj is compact, this open cover has a finite subcover.  But the Uk are nested increasing, so we must have Aj = Uk for some k, meaning Ak is empty, a contradiction.


There is a more general version [link=http://planetmath.org/?op=getobj&from=objects&id=4181]here[/link].



Powered by YaBB 1 Gold - SP 1.4!
Forum software copyright © 2000-2004 Yet another Bulletin Board